2025 AMC 10A Problems/Problem 15
Problem
In the figure below,
is a rectangle,
,
,
, and
.
What is the area of
?
Video Solution
Solution 1
Because
is a rectangle,
. We are given that
, and since
by vertical angles,
.
Let
. By the Pythagorean Theorem,
. Since
,
. Because
and
,
. By similar triangles,
. Cross-multiplying, we get that
, so
. We square both sides, and this is simply a quadratic in
:
, which has a positive root
. Since
, we can plug this into the Pythagorean Theorem, with
being the hypotenuse, to get
, and
to equal
Solution by HumblePotato, written by lhfriend,
~Corrected all incorrect side length labels and fixed typos and major errors ~Neo
Minor edit by SixthGradeBookWorm927
Minor edit by aldzandrtc
Minor edit by rcll (I lowkey fixed this shi, as aldzandrtc would have said)
Minor edit by pyraminx
Minor edit by wisewigglyjaguar
Minor edit by jeffyang2025
Solution 2 (simpler)
Draw segment
Segment
is the diagonal of rectangle
and its diagonals have length
From right triangle
we use pythagorean theorem to find
Now, we see similar triangles
and
. Let
and
We can find that
and
These triangles have a ratio of
So we get that
Cross multplying, we get
And also
Cross multiplying gives
Solving the system of equations, we find
which means
which gives
~ eqb5000/Esteban Q.
Solution 3 (thorough)
From the diagram,
and
are vertical angles and hence congruent. Additionally,
, so we have by AA Similarity that
.
Let
so
and
so
. Since the two triangles are similar, we have
. Plugging in the variables gives
.
Cross multiplying yields
.
By applying the Pythagorean Theorem on
, we get
.
Therefore,
, and plugging this back into
:
Therefore,
.
The area of
is therefore
. ~hxve
Solution 4 (Trigonometry)
Using the Pythagorean theorem, I can get
. Then, because
,
. Now, let
and
.
and
. Then, applying the sine addition formula, I get:
Thus,
, so
. This indicates that if a perpendicular is dropped from point
to the extension of line
, and the foot of the latitude is point
,
is a 3-4-5 triangle. Because
,
is also a 3-4-5 triangle. Using ratios,:
Therefore,
, so
, so
has area
, or
.
~Lollipop316
P.S. Thank you to eqb5000 and i_am_not_suk_at_math for pointing out and helping me fix typos.
Solution 5 (risky, but it works!)
Using a ruler (which is permitted during the exam), and assuming the diagram is to-scale, we can measure the physical lengths of
and
, and determine the scale factor in order to calculate
's actual math length. In my specific case (potentially could vary),
was
cm and
was between
and
cm. So, the scale with cm is
, and the length of
is around
, so the area is
. To assure ourselves that
is the most accurate estimation, we know that
is around
(too big),
even bigger (so also too big),
is just under
, and
is even bigger than
, so most likely the answer is
.
~vaishnav
Remark - You could also redraw the figure to scale. That way, you won't have to assume it's drawn to scale in the provided figure.
You would have to draw the diagonal AE, measure it as
and use pythagorean theorem to find ED as 5. Once you have that information you could sketch the whole figure to scale.
~wisegod62 (Remark and LaTeX formatting)
Solution 6 (another cheese)
Inspired by 2022 AMC 10B Problem 16, another right-triangle and rectangle mashup, we can pretend that the problem uses
similarity, because AMC problems do that a lot. In this case, we can verify it works by setting
by
similarity, so
and
. Then, we see that
, which is our only other condition, so this setup works and we can bubble in
. of course, if it weren't
, we can use any other technique.
~tiguhbabehwo
Solution 7 (area)
Connect
, which equals
. Then calculate
, which equals
.
(AA).
.
Therefore,
.
Also, let the area of
.
The area of
.
The area of
.
Solve:
So,
.
~sandpiper357
Solution 8 (analytic geometry)
As shown in the figure, establish a coordinate system with point A as the origin.
Suppose
, then the equation of
is
. Since
, the slope of
is
, the equation of
is
.
Notice that the length of
is the distance between point
and line
, due to the distance formula, we get:
The area of
is therefore
.
Solution 9
From the Pythagorean Theorem,
and we also discover
is a
right triangle. This means that
. As
by AA similarity and
, we can set up a system of linear equations in terms of
and
where they represent
and
respectively. Since
and
, we have
and
and solving the equations results in
. The area of
is
.
is unnecessary to find.
Note that
is a
triangle.
~ruihl123
Chinese Video Solution
https://www.bilibili.com/video/BV1nhkUByE3V/
~metrixgo
Video Solution (Fast and Easy to Understand)
https://youtu.be/RvU1P9qRu84?si=Ynf6wWPNB1EuF_mq ~ Pi Academy
Video Solution by SpreadTheMathLove
https://www.youtube.com/watch?v=dAeyV60Hu5c
Video Solution
~MK
Video Solution by Daily Dose of Math
~Thesmartgreekmathdude
See Also
| 2025 AMC 10A (Problems • Answer Key • Resources) | ||
| Preceded by Problem 14 |
Followed by Problem 16 | |
| 1 • 2 • 3 • 4 • 5 • 6 • 7 • 8 • 9 • 10 • 11 • 12 • 13 • 14 • 15 • 16 • 17 • 18 • 19 • 20 • 21 • 22 • 23 • 24 • 25 | ||
| All AMC 10 Problems and Solutions | ||
These problems are copyrighted © by the Mathematical Association of America.